Answer:
Yes.
Explanation:
Reactors use uranium for nuclear fuel. The uranium is processed into small ceramic pellets and stacked together onto sealed metal tubes called fuel rods. The heat created by fission turns the water into steam.
Answer:
a.) L = 2.64 kgm^2/s
b.) V = 4.4 m/s
Explanation: Jessica stretches her arms out 0.60 m from the center of her body. This will be considered as radius.
So,
Radius r = 0.6 m
Mass M = 2 kg
Velocity V = 1.1 m/s
Angular momentum L can be expressed as;
L = MVr
Substitute all the parameters into the formula
L = 2 × 1.1 × 0.6 = 1.32kgm^2s^-1
the combined angular momentum of the masses will be 2 × 1.32 = 2.64 kgm^2s-1
b. If she pulls her arms into 0.15 m,
New radius = 0.15 m
Using the same formula again
L = 2( MVr)
2.64 = 2( 2 × V × 0.15 )
1.32 = 0.3 V
V = 1.32/0.3
V = 4.4 m/s
Her new linear speed will be 4.4 m/s
Answer:
Explanation:
The amplitude of resultant wave as the result of overlap of two waves depends upon the phase difference between the two. If the waves meet crest to trough , the phase difference is 180 degree or they are in opposite phase . Hence they will destroy each other . The amplitude of resultant wave can be obtained by subtracting the amplitudes of two waves. They will interfere destructively.
Amplitude of resultant gives waves = 4.6 - 2 = 2.6 cm.
Answer and explanation:
A correct option is an option (B).
The electrical force between two charges is given as,

The electrical force is directly proportional to the product of two charges. Thus Force will depend on two charges irrespective of their signs.
Option (A) is incorrect because if charges are opposite, the value of force will not be zero. It will be -ve.
Option (C) is incorrect because the force is directly proportional to the product of charges, it depends on the amount of charge.
Option (D) is also incorrect because the force in inversly proportional to the distance between two charges. Thus, if the distance between two charges is increased, the force between two charges will decrease.
Concllusion:
The correct option is option (B).
Answer:
You are the one who will answer that because you will experiment